Math, asked by SanyaBhasin, 11 months ago

Solve the following equation:​

Attachments:

Answers

Answered by VeronicaVkhokhar
1

the answer is here in the above pic

Attachments:
Answered by prakhar157
1
here is ur answer
Mark as brainliest
Attachments:

SanyaBhasin: your answer is wrong so that's why i marked the other person as the brainliest
prakhar157: ok
SanyaBhasin: by the way thank u for your concern
Similar questions